Download as docx, pdf, or txt
Download as docx, pdf, or txt
You are on page 1of 36

STATEMENT OF FINANCIAL POSITION period. d.

The entity expects to realize the asset or intends to sell or consume it


within the entity's normal operating cycle.
Essay Questions
1. When is a "third" statement of financial position required?
4. What is the meaning of "operating cycle"?
A third statement of financial position is required when an entity:
1. Applies an accounting policy retrospectively. The operating cycle of an entity is the time between the acquisition of assets for
2. Makes retrospective restatement of items in the financial statements. processing and their realization in cash or cash equivalents. When the entity's
3. Reclassifies items in the financial statements. normal operating cycle is not clearly identifiable, its duration is assumed to be
twelve months. The operating cycle of a trading entity is the average period of time
Under these circumstances, the entity shall present three statements of that it takes for the entity to acquire the merchandise inventory, sell the inventory to
financial position as at: a. The end of the current period customers and ultimately collect cash from the sale. The operating cycle of a
b. The end of the previous period manufacturing entity is defined as the period of time between acquisition of materials
c. The beginning of the previous comparative period entering into a process and their realization in cash or cash equivalent. Thus, the
period of time to buy the materials, to convert them into finished goods, to sell the
finished goods and convert them into receivables, and to collect the receivables is
the entity's normal operating cycle.
2. Define a statement of financial position.

A statement of financial position is a formal statement showing the three elements


comprising financial position, namely assets, liabilities and equity. Investors, 5. What is the presentation of current assets in the statement of financial
creditors and other statement users analyze the statement of financial position to
evaluate such factors as liquidity, solvency and the need of the entity for additional position? Current assets are usually listed in the statement of financial
financing.
position in the order of liquidity.

3. Define current assets. The line items under current assets are:
a. Cash and cash equivalents
PAS 1, paragraph 66, provides that an entity shall classify an asset as current when: b. Financial assets at fair value such as trading securities and other investments in
quoted equity instruments.
a. The asset is cash or a cash equivalent unless the asset is restricted from being c. Trade and other receivables
exchanged or used to settle a liability for at least twelve months after the d. Inventories
reporting period. e. Prepaid expenses
b. The entity holds the asset primarily for the purpose of trading.
c. The entity expects to realize the asset within twelve months after the reporting
6. Define noncurrent assets. PAS 38, paragraph 8, defines an intangible asset simply as follows: "An intangible
asset is an identifiable nonmonetary asset without physical substance."
The caption "noncurrent assets" is a residual definition. PAS 1, paragraph 66, simply
states that "an entity shall classify all other assets not classified as current as Other noncurrent assets are those assets that do not fit into the definition of the
noncurrent". In other words, what is not included in the definition of current assets is previously mentioned noncurrent assets.
deemed excluded. All others are classified as noncurrent assets. Accordingly,
noncurrent assets include the following:
a. Property, plant and equipment 8. Define current liabilities.
b. Long-term investments
c. Intangible assets PAS 1, paragraph 69, provides that an entity shall classify a liability as current when:
d. Other noncurrent assets a. The entity expects to settle the liability within the entity's normal
operating cycle. b. The entity holds the liability primarily for the
PAS 1, paragraph 56, provides that when an entity presents current and noncurrent purpose of trading.
assets on the face of the statement of financial position, it shall not classify deferred c. The liability is due to be settled within twelve months after the reporting period. d.
tax assets as current. The entity does not have an unconditional right to defer settlement of the liability for
at least twelve months after the reporting period.

7. Define the following: Trade payables and accruals for employee and other operating costs are part of the
1. Property, plant and equipment working capital used in the entity's normal operating cycle. Such operating items are
2. Investments classified as current liabilities even if they are settled more than twelve months after
3. Intangible assets the reporting period.
4. Other noncurrent assets
Other current liabilities are not settled as part of the normal operating cycle but are
PAS 16, paragraph 6, defines property, plant and equipment as "tangible assets due for settlement within twelve months after the reporting period or held primarily
which are held by an entity for use in production or supply of goods and services, for the purpose of being traded. Examples of such current liabilities are financial
for rental to others, or for administrative purposes, and are expected to be used liabilities held for trading, bank overdraft, dividends payable, income taxes, other
during more than one period". nontrade payable and current portion of noncurrent financial liabilities. Financial
liabilities held for trading are financial liabilities that are incurred with an intention to
The International Accounting Standards Committee defines investment as follows: repurchase them in the near term.
An investment is "an asset held by an entity for the accretion of wealth through
capital distribution, such as interest, royalties, dividends and rentals, for capital
appreciation or for other benefits to the investing entity such as those obtained 9. Explain the presentation of current liabilities.
through trading relationships".
PAS 1, paragraph 54, provides that as a minimum, the face of the statement of
financial position shall include the following line items for current liabilities:
a. Trade and other payables b. An agreement to refinance or to reschedule payment on a long-term basis is
b. Current provisions completed after the reporting period and before the financial statements are
c. Short-term borrowing authorized for issue.
d. Current portion of long-term debt
e. Current tax liability However, if the refinancing on a long-term basis is completed on or before the end
of the reporting period, the refinancing is an adjusting event and therefore the
The term "trade and other payables" is a line item for accounts payable, notes obligation is classified as noncurrent. Moreover, if the entity has the discretion to
payable, accrued interest on note payable, dividends payable and accrued refinance or roll over an obligation for at least twelve months after the reporting
expenses. period under an existing loan facility, the obligation is classified as noncurrent even
No objection can be raised if the trade accounts and notes payable are if it would otherwise be due within a shorter period.

The reason for this treatment is that such obligation is considered to form part of the
separately presented. 10. What are noncurrent liabilities? entity's long-term refinancing because the entity has the unconditional right under
the existing loan agreement to defer payment for at least twelve months after the
end of the reporting period. Note that the refinancing or rolling over must be at the
The term "noncurrent liabilities" is a residual definition. PAS 1, paragraph 69, discretion of the entity. Otherwise, if the refinancing or rolling over is not at the
provides that all liabilities not classified as current liabilities are classified as discretion of the entity, the obligation is classified as a current liability.
noncurrent liabilities. 12. What are covenants?
Examples of noncurrent liabilities are:
a. Noncurrent portion of long-term debt Covenants are often attached to borrowing agreements which represent
b. Finance lease liability undertakings by the borrower. These covenants are actually restrictions on the
c. Deferred tax liability borrower as to undertaking further borrowings, paying dividends, maintaining
d. Long-term obligations to entity officers specified level of working capital and so forth.
e. Long-term deferred revenue Under these covenants, if certain conditions relating to the borrower's financial
situation are breached, the liability becomes payable on demand.
PAS 1, paragraph 56, provides that when an entity presents current and noncurrent
liabilities, it shall not classify deferred tax liabilities as current.
13. Explain the effect of breach of covenants on the classification of the liability.

11. Explain the treatment of a currently maturing long-term debt. PAS 1, paragraph 74, provides that "the liability is classified as current even if the
lender has agreed, after the reporting period and before the statements are
authorized for issue, not to demand payment as a consequence of the breach".
A liability which is due to be settled within twelve months after the reporting period is
classified as current, even if:
This liability is classified as current because at reporting date the borrower does not
a. The original term was for a period longer than twelve months.
have an unconditional right to defer payment for at least twelve months after the
reporting period. However, Paragraph 75 provides that the liability is classified as The term "reserves" is not officially defined in any accounting standard or in the
noncurrent if the lender has agreed on or before the end of reporting period to Conceptual Framework. Reserves form a substantial part of the equity of an entity.
provide a grace period ending at least twelve months after the end of reporting Under international accounting standard, the use of equity reserves is based on
period. whether a reserve is part of distributable equity or nondistributable equity.

Distributable equity is that portion that can be distributed to shareholders as


14. What is the meaning of "equity"? dividends without impairing the legal capital of the entity. This squarely pertains to
unappropriated retained earnings. Nondistributable equity is that portion that cannot
The term "equity" is the residual interest in the assets of the entity after deducting all be distributed to the shareholders in any form during the lifetime of the entity.
of its. liabilities. Simply stated, equity means "net assets" or total assets minus
liabilities. The terms used in reporting the equity of an entity depending on the form Generally, nondistributable equity reserves represent those items of equity other
of the business organization are: than the aggregate par or stated value of share capital and retained earnings
a. Owner's equity in a proprietorship unappropriated.
b. Partners' equity in a partnership Examples of reserves
c. Stockholders' equity or shareholders' equity in a corporation a. Share premium reserve is the excess over par or stated value or additional paid in
capital b. Appropriation reserve is the earmarking of retained earnings for a certain
However, the term equity may simply be used for all business organizations. purpose which may be legal, contractual or voluntary. This reserve is technicaUy
known as retained earnings appropriated
15. What are the elements of shareholders' equity? c. Asset revaluation reserve arises from the revaluation of property, plant and
equipment. It is the excess of fair value or depreciated replacement cost of the
Generally, the elements constituting shareholders' equity with their revalued property over its carrying amount. Technically, this reserve is called
equivalent IAS term are: Philippine term IAS term revaluation surplus.
Capital stock Share capital d. Other comprehensive income reserve.
Subscribed capital stock Subscribed share capital
Common stock Ordinary share capital
Preferred stock Preference share capital 17. As a minimum, what are the line items on the face of the statement of financial
Additional paid capital Share premium position?
Retained earnings (deficit) Accumulated profits (losses)
PAS 1, paragraph 54, provides that as a minimum, the face of the statement of
Retained earnings appropriated Appropriation reserve
financial position shall include the following:
Revaluation surplus Revaluation reserve
1. Cash and cash equivalents
Treasury stock Treasury share
2. Financial assets (other than 1, 3 and 6)
3. Trade and other receivables
4. Inventories
16. What is the-meaning of the term "reserves"?
5. Property, plant and equipment 19. Explain the forms of a statement of financial position.
6. Investment in associates accounted for by the equity method
7. Intangible assets The format of a statement of financial position is not specified in PAS 1.
8. Investment property
9. Biological assets In practice, there are two customary forms in presenting the statement of financial
10. Total of assets classified as held for sale and assets included in disposal group position, namely: a. Report form. This form sets forth the three major sections in a
classified as held for sale downward sequence of assets, liabilities and equity.
11. Trade and other payables b. Account form. As the title suggests, the presentation follows that of an account,
12. Current tax liability meaning, the assets are shown on the left side and the liabilities and equity on
13. Deferred tax asset and deferred tax liability the right side of the statement of financial position.
14. Provisions
15. Financial liabilities (other than 11 and 14) PAS 1, paragraph 57, provides that the standard does not prescribe the order or
16. Inabilities included in disposal group classified as held for sale format in which items are to be presented in the statement of financial position.
17. Noncontrolling interest
18. Share capital and reserves 20. Explain the presentation of assets and liabilities in the statement of financial position.

The listing of the line items is not exclusive. PAS 1, paragraph 60, provides that an entity shall present current and noncurrent
assets, and current and noncurrent liabilities on the face of the statement of
PAS 1 simply provides a list of items that are so different in nature and function to financial position. Current and noncurrent presentation of assets and liabilities
warrant separate presentation on the face of the statement of financial position. provides useful information when the entity supplies goods or
services within a clearly identifiable operating cycle.
18. When is an entity allowed to present additional line items on the face of the statement
of financial position? In the Philippines, the common practice is to present in the statement of financial
position current assets before noncurrent assets, current liabilities before
noncurrent liabilities, and equity after liabilities. Other formats may be equally
Additional line items, headings and subtotals shall be presented on the face of the
appropriate provided the distinction is clear. This is in accordance with paragraph 7
statement of financial position when such presentation is relevant to the
of the Preface to PAS 1. However, all assets and liabilities are presented broadly in
understanding of the entity's financial position.
the order of liquidity when such presentation is reliable and more relevant.
The judgment on whether additional items are presented separately is based on the
Note that the format of the statement of financial position as illustrated in the
assessment of the following:
appendix to PAS 1 presents noncurrent assets before current assets, equity before
liabilities, and noncurrent liabilities before current liabilities. This may be the practice
a. The nature and liquidity of assets. in other jurisdiction, like the United Kingdom.
b. The function of assets within the entity.
c. The amount, nature and timing of liabilities.
Multiple Choice - Theory 7. One criticism not normally aimed at a statement of financial position prepared using
current accounting and reporting standards is
Uses A. An extensive use of estimate
1. In analyzing an entity's financial statements, which financial statement would a B. Failure to include items of financial value
potential investor primarily use to assess the entity's liquidity and financial flexibility? C. Failure to reflect current value information
A. Income statement C. Statement of financial position D. The extensive use of separate classifications FA © 2014
B. Statement of cash flows D. Statement of retained earnings FA © 2014
8. Which of the following is a limitation of the statement of financial position?
2. The statement of financial position is useful for analyzing all of the A. Current fair value is not reported.
following, except A. Financial flexibility C. Profitability B. Judgment and estimate are used.
B. Liquidity D. Solvency FA © 2014 C. Many items that are of financial value are omitted.
D. All of these are considered limitation of the statement of financial position. FA © 2014
3. The statement of financial position provides a basis for all of the
following, except A. Computing rate of return. 9. In presenting a statement of financial position, an entity
B. Evaluating capital structure. A. Must make the current and noncurrent presentation.
C. Assessing liquidity and financial flexibility. B. Must present assets and liabilities in order of liquidity.
D. Determining increase in cash due to operations. FA © 2014 C. Must choose either the current and noncurrent or the liquidity presentation,
meaning free choice of presentation.
4. The amount of time that is expected to elapse until an asset is realized or otherwise D. Must make the current and noncurrent presentation, except when a presentation
converted into cash is referred to as based on liquidity provides information that is reliable and more relevant. FA © 2014
A. Exchangeability C. Liquidity
B. Financial flexibility D. Solvency FA © 2014 10. In the Philippines, the common practice is to present in the statement of financial
position A. Noncurrent assets before current assets, current liabilities before
5. The statement of financial position is useful for all of the following, except noncurrent liabilities and equity after liabilities.
A. To compute rate of return B. Noncurrent assets before current assets, noncurrent liabilities before current
B. To assess future cash flows liabilities and equity after liabilities.
C. To evaluate capital structure C. Current assets before noncurrent assets, current liabilities before noncurrent
D. To analyze cash inflows and outflows for the period TOA © 2013 liabilities and equity after liabilities.
D. Current assets before noncurrent assets, noncurrent liabilities before current
6. The statement of financial position liabilities and equity after liabilities. TOA © 2013
A. Omits many items that are of financial value
B. Uses fair value for most assets and liabilities 11. A presentation of assets and liabilities in increasing or decreasing order of liquidity
C. Makes very limited use of judgment and estimate provides information that is reliable and more relevant than a current and
D. All of the choices are correct FA © 2014 noncurrent presentation for A. Financial institution C. Public utility
B. Government-owned entity D. Service provider FA © 2014 B. II only D. Neither I nor II FA © 2014

12. An entity must present additional line items in the statement of financial position 17. Which of the following statements is true?
when such presentation is I. Biological assets should be shown in the statement of financial position.
A. Relevant to an understanding of the financial position of the entity II. The number of shares authorized for issue should be shown in the statement of
B. A generally accepted practice in the sector in which the entity operates financial position or the statement of changes in equity or in the notes.
C. Required by tax authorities of the jurisdiction in which the entity operates A. I only C. Both I and II
FA © 2014 D. Relevant to the understanding of the entity's financial position B. II only D. Neither I nor II
and financial performance
Operating cycle
13. Which of the following must be included on the face of an entity's statement of 18. It is the time between the acquisition of assets for processing and their realization in
financial position? A. Contingent liability C. Number of shares authorized FA © cash or cash equivalents.
2014 B. Investment property D. Shares in an entity owned by that entity A. Business cycle C. Cash to receivable cycle
B. Cash to inventory cycle D. Operating cycle FA © 2014
14. Which of the following must be included in an entity's statement of
financial position? A. Deferred tax 19. The operating cycle of an entity
B. Contingent asset A. Is a period of one year.
C. Share capital and reserves analyzed by class B. Is the time between the acquisition of materials entering into a process and their
D. Property, plant and equipment analyzed by class TOA © 2013 15. Which of the realization in cash or cash equivalent.
C. Causes the distinction between current and noncurrent items to depend on
following is not required to be presented as minimum information on the face of the whether they will affect cash within one year.
D. Is the period of time normally elapsed from the time the entity expends cash to the
statement of financial position? time it converts
A. Biological asset trade receivables back into cash. TOA © 2013
B. Contingent liability
C. Investment property 20. The operating cycle
D. Investment accounted under the equity method FA © 2014 A. Cannot exceed one year.
B. Refers to the seasonal variations experienced by business entities.
16. Which of the following statements in relation to the line items shown in the statement C. Should be used to classify assets and liabilities as current if it is less than one
of financial position is true? year. D. Measures the time elapsed between cash disbursement for inventory and
I. Provisions shall be recognized in the statement of financial position. cash collection of the sales price. FA © 2014
II. Liabilities included in disposal group classified as held for sale shall be shown
separately on the face of the statement of financial position. 21. The operating cycle of a business is that span of time which
A. I only C. Both I and II A. Runs from cash disbursement for inventory through the realization of cash from
sale. B. Coincides with economy's business cycle which runs from one trough of the Current & noncurrent assets
entity's business activity to the next. 26. When classifying assets as current and noncurrent for reporting purposes
C. Corresponds with its natural business year which runs from one trough of the A. The amounts at which current assets are carried and reported must reflect
particular entity's business activity to the next. realizable cash value. B. Assets are classified as current if they are reasonably
D. Is set by the industry's trade association usually on an average length of time for expected to be realized in cash or consumed during the normal operating cycle.
all entities which are members of the association. FA © 2014 C. The time period by which current assets are distinguished from noncurrent assets
is determined by the seasonal nature of the business.
22. An operating cycle D. Prepayments for items such as insurance or rent are included in "other assets"
A. Is twelve months or less in length rather than as current assets as they will ultimately be expensed. FA © 2014
B. Starts with inventory and ends with cash
C. Is the average time required for an entity to collect its receivable FA © 27. When classifying assets as current and noncurrent for reporting purposes
2014 D. Is used to determine current assets when the operating cycle is A. Prepayments are included in other assets rather than as current assets.
longer than one year B. The amounts at which current assets are reported must reflect realizable cash
value. C. Assets are classified as current if these are reasonably expected to be
23. The operating cycle concept realized in cash or consumed during the normal operating cycle.
A. Has become obsolete. D. The time period by which current assets are distinguished from noncurrent assets
B. Affects the income statement but not the statement of financial position. is determined by the seasonal nature of the business. FA © 2014
C. Causes the distinction between current and noncurrent items to depend on
whether they will affect cash within one year. 28. The basis for classifying assets as current or noncurrent is the period of time normally
D. Permits some assets to be classified as current even though they are more than elapsed from the time the accounting entity expends cash to the time it converts
one year removed from becoming cash. FA © 2014 A. Inventory back into cash, or 12 months, whichever is longer.
B. Inventory back into cash, or 12 months, whichever is shorter.
24. When there is much variability, the operating cycle is measured at C. Receivables back into cash, or 12 months, whichever is longer.
A. Less than twelve months C. The median value D. Tangible fixed assets back into cash, or 12 months, whichever is longer. FA © 2014
B. Six months D. Twelve months FA © 2014
29. Which of the following is not an acceptable major asset classification?
Current & noncurrent classification A. Current assets C. Investments FA © 2014 B. Deferred charges D.
25. Current and noncurrent presentation of assets and liabilities provides useful Property, plant, and equipment
information when the entity
A. Is a public utility Current assets
B. Is a financial institution 30. Assets to be sold, consumed or realized as part of the entity's normal
C. Is a nonprofit organization operating cycle are A. Current assets
D. Supplies goods or services within a clearly identifiable operating cycle FA © 2014 B. Noncurrent assets
C. Noncurrent investments
D. Classified as current or noncurrent in accordance with other criteria. FA © 2014 36. Which of the following should not be considered as a current asset?
A. Prepaid taxes which cover assessments of the following operating cycle of the
31. An entity shall classify an asset as current under all of the following entity. B. Installment notes receivable due over 18 months in accordance with
conditions, except A. The entity holds the asset primarily for the normal trade practice. C. Trading securities purchased by the temporary investment
purpose of trading. of cash available for current operations.
B. The entity expects to realize the asset within twelve months after the reporting D. The cash surrender value of a life insurance policy carried by an entity, the
period. C. The entity expects to realize, or intends to sell or consume the asset beneficiary, on the president. FA © 2014
within the normal operating cycle.
D. The asset is cash or cash equivalent restricted to settle a liability for more than 37. Current assets should never include
twelve months after the reporting period. FA © 2014 A. Current tax asset
B. Premium paid on a bond investment
32. The correct order to present current assets is C. Goodwill arising in a business combination
A. Cash, accounts receivable, inventories, prepaid items D. A receivable from a customer not collectible within one year FA © 2014
B. Cash, accounts receivable, prepaid items, inventories
C. Cash, inventories, accounts receivable, prepaid items 38. An entity records all sales using the installment method of accounting. Installment
D. Cash, inventories, prepaid items, accounts receivable FA © 2014 sales contracts call for 36 equal monthly cash payments. The amount of deferred
gross profit relating to collections 12 months beyond the reporting period shall be
33. Accrued revenue would normally appear in the statement of financial reported in the
position under A. Current assets C. Noncurrent assets A. Current asset section as a contra account
B. Current liabilities D. Noncurrent liabilities FA © 2014 B. Noncurrent asset section as a contra account
C. Current liability section as a deferred revenue
34. Which of the following should be classified as current asset? D. Noncurrent liability section as a deferred revenue FA © 2014
A. Cash surrender value of a life insurance policy.
B. Cash designated for the redemption of callable preference share. 39. An entity uses the installment method to recognize revenue. Customers pay the
C. Trade installment accounts receivable normally collectible in 18 months. FA © 2014 installment notes receivable in 24 equal monthly amounts which include 12%
D. A deposit on machinery ordered, delivery of which will be made within six months. interest. What is the carrying amount of the installment notes receivable six months
after the sale?
35. Which of the following should be classified as current asset? A. 75% of the original sales price
A. Cash surrender value of a life insurance policy. B. Less than 75% of the original sales price
B. Trade installment receivables normally collectible in 18 months C. The present value of the remaining monthly payments discounted at 12%
C. Cash designated for the redemption of callable preference shares FA © FA © 2014 D. Less than the present value of the remaining monthly
2014 D. A deposit on machinery ordered, delivery of which will be made payments discounted at 12%
within six months.
40. The IFRS approach in presenting current assets is
A. Inventories, accounts receivable, prepaid items, cash 46. Which of the following characteristics may result in the classification of a
B. Inventories, prepaid items, accounts receivable, cash liability as current? A. Violation of provisions of a debt agreement
C. Cash, accounts receivable, prepaid items, inventories B. Short-term obligations refinanced with long-term debt at the end of reporting
D. Cash, inventories, accounts receivable, prepaid items TOA © 2013 period C. Obligations for advance collections that involve long-term deferment of the
delivery of goods. D. Debts to be liquidated from funds that have been accumulated
41. Accrued revenue would normally appear in the statement of financial and are reported as noncurrent assets TOA © 2013
position under A. Current liabilities C. Long-term liabilities
B. Current assets D. Plant assets TOA © 2013 47. Which obligations are classified as current liabilities even if they are due to be settled
after more than twelve months from the end of the reporting period?
Current liabilities A. Bank overdrafts
42. Liabilities that are expected to be settled within the normal operating cycle B. Dividends payable
are classified as A. Equity C. Current portion of interest-bearing liabilities
B. Current liabilities D. Trade payables and accruals for employee and other operating cost FA © 2014
C. Noncurrent liabilities
D. Current or noncurrent liabilities in accordance with other criteria FA © 2014 48. A dividend declared by the entity before year-end and payable to the shareholders
three months after the end of reporting period is classified as
43. An entity shall classify a liability as current under all of the following A. A current asset C. A noncurrent liability
conditions, except A. The entity holds the liability primarily for the B. A current liability D. Equity TOA © 2013
purpose of trading.
B. The liability is due to be settled within twelve months after the reporting period. 49. At the end of reporting period, an entity has a 120-day note payable outstanding. The
C. The entity expects to settle the liability within the entity's normal operating cycle. entity has followed the policy of replacing the note rather than repaying it over the
D. The entity has an unconditional right to defer settlement of the liability for at least last three years. The entity's treasurer says that this policy is expected to continue
twelve months after the reporting period. FA © 2014 indefinitely and the arrangement is acceptable to the bank to which the note was
issued. The proper classification of the note is
44. The short-term obligations of an entity at the end of reporting period include 90-day A. Noncurrent liability
notes payable renewable for another 90-day period. The notes payable shall be B. Dependent on the actual ability to refinance
classified in the statement of financial position as C. Dependent on the intention of management
A. Current liabilities C. Intermediate debt D. Current liability, unless specific refinancing criteria are met FA © 2014 50. When an
B. Deferred charges D. Noncurrent liabilities FA © 2014
45. In which section of the statement of financial position should employment taxes that entity breaches an undertaking under a long-term loan agreement on or before the end of
are due for settlement in 15 months' time be presented?
A. Current assets C. Noncurrent assets reporting period with the effect that the liability becomes payable on demand
B. Current liabilities D. Noncurrent liabilities FA © 2014 I. The liability is classified as current even if the lender has agreed after the end of
reporting period and before the issuance of the statements not to demand
payment as a consequence of the branch. A. Goodwill C. Temporary investment FA © 2014 B. Work in process D.
II. The liability is classified as noncurrent if the lender agreed on or before the end of Accrued interest on note receivable
reporting period to provide a grace period for at least twelve months after the
end of reporting period within which to rectify the breach. 56. Which of the following items would normally be excluded from the computation of
A. I only C. Either I or II working capital? A. Prepaid insurance
B. II only D. Neither I nor II FA © 2014 B. Advances from customers
C. Cash surrender value of life insurance policy
51. Which of the following is not a current liability? D. The portion of long-term debt that matures within one year after the reporting
A. Unearned revenue period and will be paid from the regular cash account FA © 2014
B. Stock dividend payable
C. Trade accounts payable 57. An example of an item which is not an element of working capital is
D. The currently maturing portion of long-term debt FA © 2014 A. Accrued interest on note receivable C. Temporary investment
B. Goods in process D. Treasury share FA © 2014
52. An entity shall classify a liability as current when (choose the incorrect one)
A. The entity holds the liability primarily for the purpose of trading. Noncurrent assets
B. The liability is due to be settled within twelve months after the reporting period. 58. In which section of the statement of financial position should cash that is restricted for
C. The entity expects to settle the liability within the entity's normal operating cycle. the settlement of a liability due 18 months after the reporting period be presented?
D. The entity has an unconditional right to defer settlement of the liability for at least A. Current assets C. Noncurrent assets
twelve months after the reporting period. FA © 2014 B. Equity D. Noncurrent liabilities FA © 2014

Working capital 59. Which of the following is usually classified as a noncurrent asset?
53. Working capital is A. Supplies
A. Unappropriated retained earnings. B. Prepaid rent
B. Current assets less current liabilities. C. Plant expansion fund
C. Capital which has been reinvested in the business. D. Goods that are in the process of being completed for another entity FA © 2014
D. The group assets which enables the entity to operate profitably. FA © 2014
Long-term investments
54. Working capital is 60. Equity securities held to finance future construction should be classified as
A. Total current assets A. Current assets C. Long-term investments FA © 2014 B. Intangible assets
B. Capital invested in business' D. Property, plant, and equipment
C. Total current assets minus total current liabilities 61. Which of the following is not a long-term investment?
D. The group of assets which enables the entity to operate profitably FA © 2014 A. Franchise C. Land held for speculation FA © 2014 B. A sinking fund D.
Cash surrender value of life insurance
55. An example of an item which is not an element of working capital is
Noncurrent liabilities reporting period is classified as noncurrent when
62. An entity has a loan due for repayment in six months' time, but the entity has the I. An agreement to refinance or reschedule payment on a long-term basis is
option to refinance for repayment two years later. The entity plans to refinance this completed on or before the end of reporting period and before the financial
loan. In which section of the statement of financial position should this loan be statements are authorized for issue. II. The entity has the discretion to refinance or
presented? roll over the obligation for at least twelve months after the end of reporting period
A. Current assets C. Noncurrent assets under an existing loan facility.
B. Current liabilities D. Noncurrent liabilities TOA © 2013 A. I only C. Both I and II
B. II only D. Neither I nor II FA © 2014
63. Noncurrent liabilities include
A. Deferred income taxes and most lease obligations 69. A long-term debt that is due to be settled within twelve months after the end of the
B. Obligations payable at some date beyond the operating cycle reporting period is classified as noncurrent when
C. Obligations not expected to be liquidated within the operating cycle I. An agreement to refinance or reschedule payment on a long-term basis is
D. All of these FA © 2014 completed after the end of the reporting period and before the financial statements
are authorized for issue. II. The entity has the discretion to refinance or roll over the
64. Noncurrent liabilities include obligation for at least twelve months after the end of the reporting period under an
A. Deferred tax liability existing loan facility.
B. Obligations payable at some date beyond the operating cycle A. I only C. Both I and II
C. Obligations not expected to be liquidated within the operating cycle B. II only D. Neither I nor II TOA © 2013
D. All of these are noncurrent liabilities FA © 2014
70. When an entity breaches a covenant under a long-term loan agreement on or before
the end of the reporting period with the effect that the liability becomes payable on
65. Which would be classified as noncurrent liability?
demand, the liability is classified as noncurrent when
A. Unearned revenue I. The lender has agreed after the end of the reporting period and before the
B. Accrued salaries payable to management financial statements are authorized for issue not to demand payment as a
C. Mandatorily redeemable preference share consequence of the breach. II. The lender has agreed on or before the end of the
D. The currently maturing portion of long-term debt TOA © 2013 reporting period to provide a grace period ending at least twelve months after that
date.
67. An entity has a loan due for repayment in six months time, but the entity has the A. I only C. Both I and II
option to refinance for repayment two years later. The entity plans to refinance this B. II only D. Neither I nor II TOA © 2013
loan. In which section of the statement of financial position should this loan be
presented?
Shareholders’ equity
A. Current assets C. Noncurrent assets 71. As generally used, the term "net assets" represents
B. Current liabilities D. Noncurrent liabilities FA © 2014 A. Current assets less current liabilities C. Total assets less total liabilities B.
Retained earnings of an entity D. Total paid in capital of an entity FA © 2014
68. A financial liability that is due to be settled within twelve months after the end of 72. Treasury shares should be reported as
A. Current asset C. Other asset FA © 2014 B. Investment D. Reduction of B. I and III only D. I, II and III FA © 2014
shareholders' equity
Comprehensive
73. The term "deficit" refers to 78. Conceptually, asset valuation accounts are
A. A prior period error. A. Assets C. Neither assets nor liabilities
B. A debit balance in retained earnings. B. Liabilities D. Part of shareholders' equity . FA © 2014
C. An excess of current assets over current liabilities.
D. An excess of current liabilities over current assets. FA © 2014 79. Which of the following statements is true?
I. Biological assets should be shown in the statement of financial position.
74. The term "deficit" refers to II. The number of shares authorized for issue should be shown in the statement of
A. A debit balance in retained earnings. financial position or the statement of changes in equity or in the notes.
B. A loss that is reported as a prior period error. A. I only C. Both I and II
C. An excess of current assets over current liabilities. B. II only D. Neither I nor II FA © 2014
D. An excess of current liabilities over current assets. FA © 2014
80. Which of the following statements is true concerning the presentation of statement of
Comparative statements financial position?
75. An entity shall prepare how many statements of financial position as a result of I. An entity shall present current and noncurrent assets and liabilities as separate
retrospective application, retrospective restatement and reclassification of items in classification on the face of the statement of financial position.
the financial statements? A. One C. Three II. All assets and liabilities are presented broadly in the order of liquidity when such
B. Two D. Four TOA © 2013 presentation is faithfully represented and more relevant.
A. I only C. Both I and II
76. A third statement of financial position as at the beginning of the earliest comparative B. II only D. Neither I nor II FA © 2014
period is required A. When an entity applies an accounting policy retrospectively.
B. When an entity reclassifies items in the financial statements. 81. Which of the following statements is true?
C. When an entity makes a restrospective restatement of items in the I. Provisions should be recognized in the statement of financial position.
financial statements. D. In all of the above cases. TOA © 2013 II. A revaluation surplus on noncurrent assets in the current year should be
recognized in the statement of changes in equity.
77. A third statement of financial position as at beginning of the earliest comparative A. I only C. Both I and II
period presented is required B. II only D. Neither I nor II TOA © 2013
I. When an entity applies an accounting policy retrospectively.
II. When an entity makes a retrospective restatement of items in the 82. Which of the following statements is true?
financial statements. III. When an entity reclassifies items in the financial I. Dividends paid should be recognized in the statement of comprehensive
statements. income. II. A loss on disposal of assets should be recognized in the
A. I and II only C. II and III only statement of changes in equity. A. I only C. Both I and II
B. II only D. Neither I nor II TOA © 2013 Accounts receivable 1,200,000 Prepaid insurance 100,000 Financial
assets held for trading 200,000 Financial assets at fair value through
other comprehensive income 800,000 Cash 300,000 Deferred tax asset
Multiple Choice Problems 150,000 Bank overdraft 250,000
What amount should be reported as total current assets on December
Current assets 31, 2014? A. 2,100,000 C. 2,800,000
1. On December 31, 2014, the current assets of Hazel Company revealed cash and cash B. 2,550,000 D. 3,600,000 P1 © 2014
equivalents of P700,000, accounts receivable of P1,200,000 and inventories of
P600,000: The examination of accounts receivable disclosed the following: 4. Violago Company provided the following account balances on December
Trade accounts 930,000 31, 2014: Accounts receivable 1,600,000 Financial assets at fair value
Allowance for doubtful accounts ( 20,000) through profit or loss 500,000 Financial assets at amortized cost 1,300,000
Claim against shipper for goods lost in transit 30,000 Cash 1,100,000 Inventory 3,000,000 Equipment and furniture 2,500,000
Selling price of unsold goods sent by Hazel on consignment Accumulated depreciation 1,500,000 Patent 400,000 Prepaid expenses
at 130% of cost and not included in ending inventory 260,000 100,000 Equipment held for sale 1,800,000
On December 31, 2014, what total amount should be reported as
Total accounts receivable 1,200,000
current assets? A. 6,300,000 C. 8,000,000
What total amount should be reported as current assets?
B. 7,600,000 D. 8,100,000 FA © 2014
A. 2,240,000 C. 2,440,000
B. 2,412,000 D. 2,500,000 FA © 2014
5. Pamela Company provided the following adjusted account balances on
December 31,2014: Wages payable 250,000 Cash 200,000 Mortgage payable
2. Gracia Company reported the following current assets at year-end: 1,500,000 Dividends payable 150,000 Prepaid rent 100,000 Inventory 800,000
Cash including sinking fund of P500,000 with trustee 1,500,000 Sinking fund 500,000
Accounts receivable 2,500,000 Short-term investments 300,000 Investment in associate
Inventory, including P200,000 cost of goods in transit 2,000,000 Taxes payable 220,000 Accounts payable 240,000
purchased FOB point of destination 2,000,000 Accounts receivable 350,000
Advances to officers collectible currently 400,000 What total amount should be reported as current assets on December
Dividend receivable 100,000 31, 2014? A. 1,750,000 C. 3,750,000
Total current assets 6,500,000 B. 2,250,000 D. 4,250,000 P1 © 2014
What total amount should be reported as current assets?
A. 5,300,000 C. 5,800,000 6. Petite Company provided the following data on December 31,2014:
B. 5,400,000 D. 5,900,000 FA © 2014 Cash 5,000,000 Financial assets at fair value (including long-term
investment
3. Darwin Company provided the following information on December 31,2014: of P500,000 in ordinary shares of Ayala Company) 2,000,000
Inventory, including inventory expected in the ordinary course of Inventories (including goods received on
operations to be sold beyond 12 months amounting to P700,000 1,000,000 consignment of P200,000) 800,000 Prepaid expenses (including a
deposit of P50,000 made Equity of assignee in accounts receivable assigned 500,000
on inventories to be delivered in 18 months) 150,000 Property, plant, Inventory, including P600,000 cost of goods in transit
and equipment (excluding P300,000 purchased FOB destination. The goods were
of equipment still in use, but fully depreciated) 10,000,000 Goodwill received on January 3, 2015 2,800,000
1,000,000 Total assets 18,950,000 What total amount of current assets should be reported on December
31,2014? A. 7,400,000 C. 7,900,000
Cash in general checking account 3,500,000 Cash in fund to be used to B. 7,700,000 D. 8,000,000 P1 © 2014
retire bonds in 2016 1,000,000 Cash held to pay value added taxes
500,000 Total cash 5,000,000 9. Caticlan Company provided the following data on December 31,2014:
What total amount of current assets should be reported on December Cash, including sinking fund of P500,000 for bond
31, 2014? A. 6,200,000 C. 7,200,000 payable due on June 30, 2015 2,000,000 Notes receivable 1,200,000
B. 6,250,000 D. 7,250,000 P1 © 2014 Note receivable discounted 700,000 Accounts receivable - unassigned
3,000,000
7. Arabian Company reported the following current assets on December Accounts receivable - assigned 800,000 Equity of assignee in accounts
31,2014: Cash 4,300,000 Accounts receivable 7,500,000 Inventory receivable assigned 500,000 Inventory, including P600,000 cost of
4,000,000 goods in transit purchased
Deferred tax asset 1,200,000 FOB destination. The goods were received on Jan. 3, 2015 2,800,000
17,000,000 Allowance for doubtful accounts 100,000 What total amount of current
An analysis of the accounts receivable disclosed that accounts receivable comprised assets should be reported on December 31, 2014? A. 7,400,000 C.
the following: Trade accounts receivable 5,000,000 Allowance for doubtful 7,900,000
accounts ( 500,000) Selling price of Arabian Company's unsold goods sent B. 7,700,000 D. 8,400,000 FA © 2014
to Tar Company on consignment at 150% of cost
and excluded from Arabian's ending inventory 3,000,000 7,500,000 On 10. On December 31, 2014, Ivan Company showed the following current assets:
December 31, 2014, what amount should be reported as total current Cash 500,000 Accounts receivable 2,500,000 Inventory 2,000,000
assets? A. 14,800,000 C. 15,800,000 Prepaid expenses 100,000 Total current assets 5,100,000 Cash on
B. 15,300,000 D. 16,000,000 P1 © 2014 hand including customer's postdated check
of P20,000 and employee IOU of PI0,000 130,000 Cash in bank per
8. Caticlan Company provided the following data on December 31,2014: bank statement (outstanding
Cash, including sinking fund of P500,000 2,000,000 checks on December 31, 2014, P70,000) 370,000 Total cash 500,000
Notes receivable 1,200,000 Customers' debit balances, net of customers' deposit of P50,000
Note receivable discounted 700,000 1,900,000 Allowance for doubtful accounts (150,000) Sales price of
Accounts receivable - unassigned 3,000,000 goods invoiced to customers at 150% of cost on
Accounts receivable - assigned 800,000 December 29, 2014 but delivered on January 5, 2015 and
Allowance for doubtful accounts 100,000 excluded from reported inventory 750,000 Total accounts receivable
2,500,000 What total amount of current assets should be reported?
A. 4,630,000 C. 4,830,000 Accounts receivable 1,500,000
B. 4,780,000 D. 4,900,000 P1 © 2014 Allowance for doubtful accounts 200,000
Inventory 2,000,000
11. East Company reported the following current assets at year end: Prepaid insurance 300,000
Cash 3,200,000 Accounts receivable 2,000,000 Inventory The cash included cash in bank of P1,000,000 net of bank overdraft P300,000, cash
2,800,000 set aside for plant purchase P2,000,000, petty cash P10,000, cash withheld from
Deferred charges 200,000 wages P190,000, and general cash P500,000.
8,200,000 The accounts receivable included past due account in the amount of P100,000. The
The accounts receivable consisted of the following: account is deemed uncollectible and should be written off. The inventory included
Customers' accounts 1,420,000 goods held on consignment amounting to P150,000 and goods of P200,000
Employees' account-current 240,000 purchased and received on December 31,2014. Neither of these items have been
Advances to subsidiary 260,000 recorded as a purchase. The prepaid insurance included cash surrender value of
Allowance for uncollectible accounts (120,000) life insurance of P50,000. What total amount should be reported as current assets
Claim against shipper for goods lost in transit 200,000 on December 31, 2014?
2,000,000 A. 5,100,000 C. 5,300,000
What amount should be reported as total current assets? B. 5,200,000 D. 5,400,000 P1 © 2014
A. 7,740,000 C. 7,940,000
B. 7,780,000 D. 8,200,000 FA © 2014 14. Gold Company provided the following trial balance on December 31,2014:
Cash overdraft 100,000
12. The general ledger trial balance of Daria Company included the following accounts Accounts receivable, net 350,000
on December 31, 2014: Inventory 580,000
Inventory, including inventory expected in the ordinary course Prepaid expenses 120,000
of operations to be sold beyond 12 months amounting to P700.000 Land classified as held for sale 1,000,000
1,000,000 Trade receivables 1,200,000 Prepaid insurance 100,000 Property, plant and equipment, net 950,000
Financial assets held for trading 200,000 Financial assets at fair value Accounts payable and accrued expenses 320,000
through other comprehensive income 800,000 Cash 300,000 Deferred Ordinary share capital 250,000
tax asset 150,000 Bank overdraft 250,000 What total amount should be Share premium 1,500,000
reported as current assets on December 31, 2014? Retained earnings . 830,000
A. 2,100,000 C. 2,800,000 3,000,000 3,000,000
B. 2,550,000 D. 3,600,000 FA © 2014 Checks amounting to P300,000 were written to vendors and recorded on December
29,2014, resulting in a cash overdraft of P100,000. The checks were mailed on
13. Daet Company provided the following account balances and related information on January 15,2015. Land classified as held for sale was sold for cash on January
December 31,2014: 31,2015. The entity issued the financial statements on March 31, 2015. On
Cash 3,700,000 December 31,2014, what total amount should be reported as current assets?
A. 1,250,000 C. 2,050,000 January 5, 2015 and excluded
B. 1,950,000 D. 2,250,000 P1 © 2014 from reported inventory 750,000
Subscription receivable, collectible currently 1,000,000
15. Petite Company provided the following data on December 31, 2014: Total accounts receivable 3,500,000
Cash 5,000,000 What total amount should be reported as current assets on December 31, 2014?
Financial assets at fair value (including long-term A. 5,800,000 C. 5,900,000
investment of P500,000 in ordinary shares of Ayala Company) B. 5,830,000 D. 6,230,000 FA © 2014
2,000,000 Inventories (including goods received on consignment of
P200,000) 800,000 Prepaid expenses (including a deposit of 17. Daet Company provided the following account balances and related information on
P50,000 made on December 31, 2014:
inventories to be delivered in 18 months) 150,000 Property, plant, Cash and cash equivalents 3,700,000
and equipment (excluding P300,000 of equipment Accounts receivable 1,500,000
still in. use but fully depreciated) 10,000,000 Goodwill (based on Allowance for doubtful accounts ( 200,000)
estimate by the president) 1,000,000 Total assets 18,950,000 Analysis Inventory 2,000,000
of the cash account showed the following: Prepaid insurance 300,000
Cash in general checking account 3,500,000 Sinking fund set aside 7,300,000
to retire bonds in 2016 1,000,000 Cash held to pay value added The cash and cash equivalents included the following:
taxes 500,000 5,000,000
Cash in bank, net of bank overdraft of P300,000
What total amount of current assets should be reported on
maintained in a separate bank 1,000,000 Cash set aside by the
December 31, 2014? A. 6,200,000 C. 7,200,000 Board of Directors for the purchase of a plant site 2,000,000 Petty
B. 6,250,000 D. 7,250,000 FA © 2014 cash 10,000 Cash withheld from wages for income tax of employees
190,000 General cash 500,000 3,700,000
16. On December 31, 2014, Ivan Company showed the following current * The accounts receivable included past due account in the amount of P100,000 on
assets: Cash 500,000 Accounts receivable 3,500,000 which a loss of 50% is anticipated. The account should be written off.
Inventory 2,000,000 Deferred tax asset 400,000 Prepaid * The merchandise inventory included goods held on consignment amounting to
expenses 100,000 Total current assets 6,500,000 Cash on P150,000 and goods of P200,000 purchased and received on December 31,
hand including customer's postdated 2014. Neither of these items had been recorded as a purchase.
check ofP20,000 and employee IOU of P10,000 130,000 * The prepaid insurance included cash surrender value of life insurance
Cash in bank per bank statement (outstanding of P50,000. What total amount should be reported as current assets
checks on December 31, 2014, P70,000) 370,000 Total cash on December 31, 2014?
500,000 Customers' debit balances, net of customers' A. 5,100,000 C. 5,300,000
deposit of P50,000 1,900,000 Allowance for doubtful
B. 5,200,000 D. 5,400,000 FA © 2014
accounts ( 150,000) Sales price of goods invoiced to
customers at
Total assets
150% of cost on December 29, 2014 but delivered on
18. Rice Company was incorporated on January 1, 2014 with P5,000,000 from the A. 9,000,000 C. 19,000,000
issuance of share capital and borrowed funds of PI,500,000. During the first year, B. 15,000,000 D. 21,500,000 FA © 2014
net income was P2,500,000. On December 15, the entity paid a P500,000 cash
dividend. On December 31,2014, the liabilities had increased to P1,800,000. On 22. Mill Company reported the following account balances on December
December 31,2014, what amount should be reported as total assets? 31, 2014: Accounts payable 1,500,000
A. 6,500,000 C. 8,800,000 Bonds payable, due 2015 2,500,000
B. 6,800,000 D. 9,300,000 P1 © 2014 Discount on bonds payable 300,000
Dividend payable 800,000
19. Mirr Company was incorporated on January 1,2014 with proceeds from the issuance Note payable, due 2016 2,000,000
of P7,500,000 in share capital and borrowed funds of PI, 100,000. During the first What total amount should be reported as current liabilities?
year, revenue from sales and consulting amounted to P8,200,000, and operating A. 4,500,000 C. 6,500,000
costs and expenses totaled P6,400,000. On December 15,2014, the entity declared B. 5,100,000 D. 7,800,000 FA © 2014
a P300,000 dividend, payable to shareholders on January 15,2015. The liabilities
increased to P2,000,000 by December 31, 2014. On December 31,2014, what
23. Brite Company had the following information on December 31,2014:
amount should be reported as total assets?
Accounts payable 550,000
A. 10,100,000 C. 11,300,000
Unsecured note payable, 8%, due July 1, 2015 4,000,000
B. 11,000,000 D. 12,100,000 P1 © 2014
Accrued expenses 350,000
Contingent liability 450,000
20. Peach Company reported total assets of P4,375,000 at year-end which
included the following: Treasury shares of Peach Company at cost 120,000 Deferred tax liability 250,000
Unamortized patent 56,000 Senior bonds payable, 7%, due March 31,2015 5,000,000
Cash surrender value of life insurance 68,500 What amount should be reported as total current liabilities?
Cumulative translation loss 42,000 A. 4,900,000 C. 10,150,000
What amount should be reported as total assets at year-end? B. 9,900,000 D. 10,350,000 P1 © 2014
A. 4,208,500 C. 4,250,500
24. Gar Company reported the following liability account balances on
B. 4,213,000 D. 4,255,000 FA © 2014
December 31,2014: Accounts payable 1,900,000
Bonds payable 3,400,000
Current liabilities
Premium on bonds payable 200,000
21. Brazil Company reported the following liability balances on December 31, 2014:
Accounts payable 5,000,000 Bonds payable, due December 30, 2015 10,000,000 Deferred tax liability 400,000
Deferred tax liability 2,500,000 Note payable-bank 4,000,000 The bank note Dividends payable 500,000
payable matures on June 30, 2015. On March 1, 2015, the bank note payable was Income tax payable 900,000
refinanced on a long-term basis. The financial statements were issued on March 31, Note payable, due January 31,2015 600,000
2015. What total amount should be reported as current liabilities? The deferred tax liability is based on temporary differences that will reverse in
2016. On December 31,2014, what total amount should be reported as current
liabilities? payable should be classified as current on December 31, 2014?
A. 3,900,000 C. 4,300,000 A. 0 C. 4,000,000
B. 4,100,000 D. 7,100,000 FA © 2014 B. 2,000,000 D. 6,000,000 P1 © 2014

25. Burma Company disclosed the following information: 28. Jam Company had P2,000,000 note payable due on March 1, 2015. The entity
Accounts payable, after deducting debit balances borrowed P1,500,000 on December 31, 2014 which has a five-year term and used
in suppliers' accounts amounting to P100,000 4,000,000 Accrued the proceeds to pay down the note payable and used other cash to pay the balance
expenses 1,500,000 Credit balances of customers' accounts 500,000 at maturity. The financial statements were issued on March 31, 2015. What amount
Stock dividend payable 1,000,000 Claims for increase in wages and of the note payable should be classified as current on December 31, 2014?
allowance by A. 0 C. 1,500,000
employees of the entity, covered in a pending lawsuit 400,000 B. 500,000 D. 2,000,000 FA © 2014
Estimated expenses in redeeming prize coupons 600,000 What amount
should be reported as total current liabilities? 29. On December 31,2014, Ace Company had P40,000,000 note payable due on
A. 6,600,000 C. 7,100,000 February 28,2015. On December 31, 2014, the entity arranged a line of credit with
B. 6,700,000 D. 7,700,000 FA © 2014 City Bank which allows the entity to borrow up to P35,000,000 at one percent above
the prime rate for three years. On February 15,2015, the entity borrowed
26. Gumamela Company provided the following data at year-end: P25,000,000 from City Bank and used P5,000,000 additional cash to liquidate
Trade accounts payable, including cost of goods P30,000,000 note payable. The financial statements were issued on March 31,2015.
received on consignment of P150,000 1,350,000 Accrued taxes What amount of note payable should be reported as current liability on December
payable 125,000 Customers' deposit 100,000 Manila Company as 31,2014?
guarantor 200,000 Bank overdraft 55,000 Accrued electric and power A. 0 C. 10,000,000
bills 60,000 Reserve for contingencies 150,000 What total amount B. 5,000,000 D. 40,000,000 FA © 2014
should be reported as current liabilities?
A. 1,540,000 C. 1,740,000 30. Charice Company provided the following information on December 31,2014: *
B. 1,650,000 D. 1,840,000 FA © 2014 Accounts payable amounted to P500,000 and accrued expenses totaled P300,000
on December 31,2014.
27. Mazda Company reported the following liability balances on December * On December 15,2014, die entity declared a cash dividend of P7 per share on
31,2014: 10% note payable issued on October 1,2013, maturing 100,000 outstanding shares, payable on January 15,2015.
October 1, 2015 2,000,000 12% note payable issued on March 1, * On July 1, 2014, the entity issued P5,000,000, 8% bonds for P4,400,000 to yield
2013, maturing on March 1, 2015 4,000,000 10%. The bonds mature on June 30, 2019, and pay interest annually every
The 2014 financial statements were issued on March 31,2015. Under the loan June 30.
agreement for the 10% note payable, the entity has the discretion to refinance the * The pretax financial income was P8,500,000 and taxable income was P6,000,000.
obligation for at least twelve months after December 31, 2014. On March 1, 2015, The difference is due to P1,000,000 permanent difference and PI,500,000 of
the entire P4,000,000 balance of the 12% note payable was refinanced through taxable temporary difference to
issuance of a long-term obligation payable lump sum. What amount of the notes reverse in 2015.
* The income tax rate is 30%. The entity made estimated income tax payments * The 10% mortgage note was issued October 1,2011, with a term of 10 years.
during the year of P1,000,000. Terms of the note give the holder the right to demand immediate payment if
What amount should be reported as total current liabilities on the entity fails to make a monthly interest payment within 10 days from the
December 31,2014? A. 2,300,000 C. 2,700,000 date the payment is due. On December 31,2014, the entity is three months
B. 2,500,000 D. 3,500,000 P1 © 2014 behind in making the required interest payment.
* The bonds payable are ten-year, 8% bonds, issued June 30,2005. Interest is
31. Manchester Company provided the following information on December payable semiannually on June 30 and December 31.
31,2014: Employee income taxes withheld 900,000 Cash balance at What amount should be reported as total current liabilities?
First State Bank 2,500,000 Cash overdraft at Harbor Bank 1,300,000 A. 11,650,000 C. 15,650,000
Accounts receivable with credit balance 750,000 Estimated expenses B. 13,650,000 D. 20,650,000 FA © 2014
of meeting warranties 500,000 Estimated damages as a result of
unsatisfactory performance on a contract1,500,000 Accounts payable 33. Charice Company provided the following information on December 31, 2014: *
3,000,000 Deferred serial bonds, issued at par and bearing interest at Accounts payable for goods and services purchased on open account amounted to
12%, P500,000 and accrued expenses totaled P300.000 on December 31, 2014.
payable in semiannual installments of P500,000 due April 1 and * On December 15, 2014, the entity declared a cash dividend of P7 per share,
October 1 of each year, the last bond to be paid on October 1, 2020. payable on January 15, 2015, to shareholders of record on December 31, 2014.
Interest is also paid semiannually. 5,000,000 What amount should The entity had 100,000 shares issued and outstanding throughout 2014.
be reported on December 31,2014 as total current liabilities? A. * On July 1, 2014, the entity issued P5,000,000, 8% bonds for P4,400,000 to yield
7,350,000 C. 8,100,000 10%. The bonds mature on June 30, 2019, and pay interest annually every June
B. 7,950,000 D. 9,100,000 P1 © 2014 30. On December 31, 2014, the bonds were trading in the open market at 86 to
yield 12%. The entity used the effective interest method to amortize bond
32. Kumaykay Company provided the following information on December discount.
31,2014: Accounts payable 6,500,000 * The pretax financial income was P8,500,000 and taxable income was P6,000,000.
Bank note payable - 10% 3,000,000 The difference is due to PI,000,000 permanent difference and Pi,500,000 of
Bank note payable - 11% 5,000,000 taxable temporary difference which is expected to reverse in 2015. The entity is
Interest payable 150,000 subject to income tax rate of 30% and made estimated income tax payments
Mortgage note payable - 10% 2,000,000 during the year of P1,000,000.
Bonds payable 4,000,000 What total amount should be reported as current liabilities on
* The P3,000,000, 10% note was issued March 1,2014, payable on demand. December 31, 2014? A. 2,300,000 C. 2,700,000
Interest is payable every six months. B. 2,500,000 D. 3,500,000 FA © 2014
* The one-year P5,000,000,11% note was issued January 15,2014. On
December 31,2014, the entity negotiated a written agreement with the bank to 34. Manchester Company provided the following information on December 31,
replace the note with a 2-year, 2014: Employee income taxes withheld 900,000 Cash balance at First State
P5,000,000,10% note to be issued January 15,2015. Bank 2,500,000 Cash overdraft at Harbor Bank 1,300,000 Accounts
receivable with credit balance 750,000
Estimated expenses of meeting warranties on Vacant land held as investment 5,000,000
merchandise previously sold 500,000 Estimated damages as a Equipment classified as held for sale 200,000
result of unsatisfactory Accounts payable 1,400,000
performance on a contract 1,500,000 Accounts payable 3,000,000 Note payable - due 2015 800,000
Deferred serial bonds, issued at par and bearing interest at 12%, Note payable -due 2016 300,000
payable in semiannual installments of P500,000 due April 1 Bonds payable - due 2017 4,600,000
and October 1 of each year, the last bond to be paid on Salaries payable 400,000
October 1, 2020. Interest is also paid semiannually. 5,000,000 Stock Value added taxes payable 300,000
dividend payable 2,000,000 On December 31, 2014, what total amount What is the working capital on December 31, 2014?
should be reported as current liabilities? A. 7,350,000 C. 8,100,000 A. 1,700,000 C. 2,260,000
B. 7,950,000 D. 9,100,000 FA © 2014 B. 2,000,000 D. 4,900,000 P1 © 2014
Working capital 37. Aroma Company provided the following information on December 31,
35. Alena Company provided the following information at year-end: 2014: Cash 300,000
Property, plant and equipment 35,000,000 Land 20,000,000 Cash Accounts receivable, net of allowance of P50,000 800,000
5,000,000 Accounts receivable 20,000,000 Allowance for doubtful Inventory 1,650,000
accounts 1,000,000 Merchandise inventory 13,000,000 Prepaid Prepaid expenses 250,000
insurance 2,500,000 Financial asset at fair value through other Property, plant and equipment 8,800,000
comprehensive income 7,000,000 Accounts payable 8,000,000 Wages
Accumulated depreciation 800,000
payable 2,000,000 Short-term note payable 3,000,000 Bonds payable
40,000,000 Premium on bonds payable 3,000,000 What is the working Accounts payable 1,250,000
capital? Accrued expenses 250,000
A. 26,500,000 C. 35,500,000 Bonds payable 4,000,000
B. 33,500,000 D. 46,500,000 P1 © 2014 36. Moon Company provided the Share capital 5,000,000
Retained earnings 500,000
following data on December 31,2014. A P500,000 note payable to bank, due on June 30, 2015, was deducted from the
balance on deposit in the same bank. The entity recorded checks of P200,000 in
Cash in general checking account 500,000 payment of accounts payable on December 31,2014. These checks were still on
Sinking fund to be used to retire bonds in 2017 1,200,000 hand on January 20, 2015. An advance payment of
P100,000 from a customer for goods to be delivered in 2015 was deducted from
Cash held to pay value added taxes 300,000
accounts receivable. What is the working capital on December 31,2014?
Note receivable - due February 2016 2,200,000
A. 1,400,000 C. 2,000,000
Accounts receivable 2,100,000
B. 1,500,000 D. 3,800,000 P1 © 2014
Inventory 1,500,000
Prepaid insurance 300,000
Noncurrent liabilities
38. Jam Company had P2,000,000 note payable that is due on February 28, 2015. The Total shareholders’ equity
entity borrowed PI,600,000 on February 25, 2015 which has afive year term and 41. Magnolia Company reported the following unadjusted current assets and
used the proceeds to pay down the note and used other cash to pay the balance. shareholders' equity on December 31, 2014:
How much of the P2,000,000 note is classified as noncurrent in the December Cash 150,000 Investments in trading equity securities (including
31,2014 financial statements that were issued on March 31, 2015? P750,000 cost of Magnolia Company shares) 1,000,000 Trade
A. 0 C. 1,600,000 accounts receivable 850,000 Inventories 370,000 Total current
B. 400,000 D. 2,000,000 P1 © 2014 assets 2,370,000 Share capital 5,000,000 Retained earnings
1,000,000 Total shareholders' equity 6,000,000
39. Mazda Company reported the following liability balances on December 31, The investments and inventories are reported at costs which approximate market
2014: 10% note payable issued on October 1, 2013 maturing October value. What amount should be reported as total shareholders' equity on December
1,2015 2,000,000 12% note payable issued on March 1, 2013 maturing 31, 2014?
on March 1, 2015 4,000,000 A. 5,000,000 C. 6,000,000
The 2014 financial statements were issued on March 31, 2015. The entity has the B. 5,250,000 D. 6,750,000 FA © 2014
discretion to refinance the 10% note payable for at least twelve months after
December 31, 2014. On December 31, 2014, the entire P4,000,000 balance of the 42. United Company provided the following current assets and shareholders' equity on
12% note payable was refinanced on a long-term basis. What amount of the notes December 31,2014:
payable should be classified as noncurrent on December 31, 2014? Cash 600,000
A. 0 C. 4,000,000 Financial assets at fair value (including cost of
B. 2,000,000 D. 6,000,000 FA © 2014 P300,000 of United Company's shares) 1,000,000
Accounts receivable 3,500,000
40. Cara Company provided the following information: Inventory 1,500,000
1/1/2014 12/31/2014 Total current assets 6,600,000
Current assets 240,000 ?
Property, plant, and equipment 1,600,000 1,700,000 Share capital 5,000,000
Current liabilities ? 130,000 Share premium 2,000,000
Noncurrent liabilities 580,000 ? Retained earnings 500,000
All assets and liabilities are reported at year-end. Working capital of P92,000 Total shareholders' equity 7,500,000
remained unchanged from 2013 to 2014. Net income in 2014 was P64,000. No What amount should be reported as total shareholders' equity?
dividends were declared during 2014 and there were no other changes in A. 5,200,000 C. 7,500,000
shareholders' equity. What amount should be reported as noncurrent liabilities on B. 7,200,000 D. 7,800,000 P1 © 2014
December 31, 2014?
A. 340,000 C. 580,000 43. Kalinga Company provided the following information at year-end:
B. 432,000 D. 616,000 FA © 2014 Share capital 15,000,000
Share premium 5,000,000
Treasury shares, at cost 2,000,000 reported net assets totaling P8,750,000. Included in the asset section were the
Actuarial loss on defined benefit plan 1,000,000 following:
Retained earnings unappropriated 6,000,000 Treasury shares of Mont Company at cost, which approximates
Retained earnings appropriated 3,000,000 market value on December 31 250,000 Idle machinery 100,000
Revaluation surplus 4,000,000 Cash surrender value of life insurance 150,000 Allowance for inventory
Cumulative translation adjustment-credit 1,500,000 write-down 50,000 What amount should be reported as net assets?
What amount should be reported as total shareholders' equity? A. 8,350,000 C. 8,500,000
A. 25,500,000 C. 31,500,000 B. 8,450,000 D. 9,000,000 FA © 2014
B. 28,500,000 D. 32,500,000 P1 © 2014
Comprehensive
44. Silver Company provided the following information at year-end: Current assets & current liabilities
Share premium 1,000,000 Questions 1 & 2 are based on the following information. FA © 2014 Kabugao
Accounts payable 1,100,000 Company provided the following information on December 31,2014:
Preference share capital, at par 2,000,000 Cash in bank, net of bank overdraft of P500,000 5,000,000 Petty cash
Ordinary share capital, at par 3,000,000 (unreplenished petty cash expenses, P10,000) 50,000 Notes receivable
4,000,000 Accounts receivable, net of accounts with credit balances of
Sales 10,000,000
P1,500,000 6,000,000 Inventory 3,000,000 Bond sinking fund 3,000,000
Total expenses 7,800,000 Total current assets 21,050,000
Treasury shares at cost - ordinary 500,000
Dividends 700,000 Accounts payable, net of suppliers' accounts with debit balances of
Retained earnings - January 1 1,000,000 P1,000,000 7,000,000 Notes payable 4,000,000 Bond payable due June
What total shareholders' equity should be reported on December 31 ? 30, 2015 3,000,000 Accrued expenses 2,000,000 Total current liabilities
A. 5,800,000 C. 8,500,000 16,000,000
B. 8,000,000 D. 8,700,000 P1 © 2014
47. What amount should be reported as total current assets on December 31,2014?
45. Mont Company reported net assets totaling P8,750,000 at year-end which A. 19,040,000 C. 20,050,000
included the following: Treasury shares of Mont Company at cost 250,000 B. 20,040,000 D. 24,040,000
Idle machinery 100,000
Trademark 150,000 48. What amount should be reported as total current liabilities on
Allowance for inventory write-down 200,000 December 31,2014? A. 15,000,000 C. 16,000,000
What amount should be reported as net assets at year-end? B. 15,500,000 D. 19,000,000
A. 8,200,000 C. 8,400,000
B. 8,300,000 D. 8,500,000 P1 © 2014 Questions 1 & 2 are based on the following information. FA © 2014 Camarines
Company provided the following data on December 31, 2014:
46. When preparing a draft of year-end statement of financial position, Mont Company
Cash 2,000,000 Accounts receivable 3,000,000 Inventory adjustment 400,000
1,900,000 Prepaid expenses 100,000 Accounts payable, net of Revenue 3,600,000 Expenses 2,600,000 . 5,550,000 5,550,000
debit balance of P50,000, 2,450,000 Interest payable 150,000 During 2014, estimated tax payments of P300,000 were charged to prepaid taxes. The
Income tax payable 300,000 Money claim of the union pending entity has not yet recorded income tax expense. There were no differences between
final decision 500,000 Mortgage payable, due in four annual financial and taxable income. The tax rate is 30%.
installments 2,000,000
The cash of P2,000,000 included cash in bank of Pl,650,000, a customer check of Included in accounts receivable is P500,000 due from a customer. Special terms granted
PI00,000 marked NSF, an employee IOU of P50,000 and P200,000 deposited with the to this customer require payment in equal semiannual installments of P 125,000 every
court for a case under litigation. The cash in bank of P1,650,000 is the balance per bank April 1 and October 1.
statement. On December 31,2014, outstanding checks amounted to P250,000.
The accounts receivable included the following: 51. On December 31,2014, what amount should be reported as total
Customers' debit balances 1,600,000 Advances to subsidiary current assets? A. 2,000,000 C. 2,250,000
400,000 Advances to suppliers 200,000 Receivable from officers B. 2,200,000 D. 2,300,000
300,000 Allowance for doubtful accounts (100,000) Selling price
of merchandise invoiced at 120% 52. On December 31,2014, what amount should be reported as total
of cost, undelivered and excluded from inventory 600,000 retained earnings? A. 1,200,000 C. 1,630,000
B. 1,330,000 D. 1,680,000
49. What amount should be reported as total current assets?
A. 5,550,000 C. 6,100,000 Questions 1 & 2 are based on the following information.
B. 6,050,000 D. 6,350,000 The following trial balance of Trey Company on December 31, 2014 has been adjusted
except for income tax expense.
50. What amount should be reported as total current liabilities? Cash 5,000,000
A. 3,400,000 C. 3,700,000 Accounts receivable, net 8,000,000
B. 3,450,000 D. 3,950,000 Prepaid taxes 1,500,000
Inventory 6,000,000
Current assets & retained earnings Property, plant and equipment 17,000,000
Questions 1 & 2 are based on the following information. P1 © 2014 Trey Company Accounts payable 10,000,000 Share capital 20,000,000 Retained
provided the following trial balance on December 31. 2014 which had been adjusted earnings 5,000,000 Foreign currency translation adjustment
except for income tax expense: 2,500,000
Cash 600,000 Revenue 15,000,000 Expenses 10,000,000 . 50,000,000
Accounts receivable, net of allowance of P100,000 1,650,000 50,000,000
Prepaid taxes 300,000 During 2014, estimated tax payments of PI,500,000 were charged to prepaid taxes.
Accounts payable 140,000 Share capital 500,000 Share premium The entity has not yet recorded income tax expense. The tax rate is 30%. The
680,000 Retained earnings 630,000 Foreign currency translation accounts receivable included P3,000,000 due from a customer. Special terms
granted to this customer require payment in equal semiannual installments of 56. What total amount should be reported as shareholders' equity?
P500,000 every April 1 and October 1. A. 1,330,000 C. 2,580,000
B. 1,750,000 D. 2,900,000
53. On December 31, 2014, what total amount should be reported as
current assets? A. 17,000,000 C. 19,500,000 Current assets, current liabilities, retained earnings
B. 18,500,000 D. 21,000,000 Questions 1 thru 3 are based on the following information. P1 © 2014 Shaw Company
provided the following trial balance on December 31, 2014 which had been adjusted
54. On December 31, 2014, what amount should be reported as total except for income tax expense:
retained earnings? A. 5,750,000 C. 8,500,000 Cash 675,000
B. 6,000,000 D. 10,000,000 Accounts receivable (net) 2,695,000
Inventory 2,185,000
Current assets & shareholders’ equity Property, plant and equipment (net) 10,245,000
Questions 1 & 2 are based on the following information. Accounts payable and accrued liabilities 1,800,000 Income tax
Gold Company provided the following trial balance on June 30,2014: payable 1,500,000
Cash overdraft 100,000 Deferred tax liability 750,000 Share capital 2,500,000 Share
Accounts receivable 350,000 premium 3,000,000
Inventory 580,000 Retained earnings, January 1 3,350,000 Net sales and other
Prepaid expenses 120,000 revenue 15,000,000 Costs and expenses 10,000,000
Land classified as held for sale 1,000,000 Income tax expense 2,100,000 .
Property, plant and equipment, net 950,000 27,900,000 27,900,000
Accounts payable and accrued expenses 320,000
Share capital 250,000 The accounts receivable included P1,000,000 due from a customer and payable in
Share premium 1,500,000 quarterly installments of P125,000. The last payment is due December 30,2016. During
Retained earnings . 830,000 the year, estimated tax payment of P600,000 was charged to income tax expense. The
3,000,000 3,000,000 income tax rate is 30%o on all types of income.
Checks amounting to P300,000 were written to vendors and recorded on June 29, 2014
resulting in a cash overdraft of P100,000. The checks were mailed on July 9, 2014. Land On December 31,2014, what amount should be reported as
classified as held for sale was sold for cash on July 15, 2014. The entity issued the
financial statements on July 31, 2014. 57. Total current assets?
A. 5,055,000 C. 5,555,000
55. What total amount should be reported as current assets? B. 5,530,000 D. 6,030,000
A. 1,250,000 C. 2,050,000
B. 1,950,000 D. 2,250,000 58. Total current liabilities?
A. 2,700,000 C. 3,450,000
B. 3,300,000 D. 4,050,000 A. 6,250,000 C. 7,750,000
B. 6,850,000 D. 8,350,000
59. Retained earnings?
A. 6,250,000 C. 7,750,000 Questions 1 thru 3 are based on the following information.
B. 6,850,000 D. 8,350,000 Mint Company provided the following account balances on December 31, 2014 which
had been adjusted except for income tax expense:
Questions 1 thru 3 are based on the following information. Cash 600,000 Accounts receivable, net 3,500,000 Cost in excess
Shaw Company provided the following trial balance on December 31, 2014 which had of billings on long-term contracts 1,600,000 Billings in excess of
been adjusted except for income tax expense: cost on long-term contracts 700,000
Cash 675,000 Accounts receivable 2,695,000 Inventory Prepaid taxes 450.000 Property, plant, and equipment, net
2,185,000 1,510,000
Property, plant and equipment 10,245,000 Accounts payable and Note payable - noncurrent 1,620,000 Share capital 750,000
accrued liabilities 1,800,000 Income tax payable 1,500,000 Share premium 2,030,000 Retained earnings unappropriated
Deferred tax liability 750,000 900,000
Share capital 2,500,000 Share premium 3,000,000 Retained Retained earnings restricted for note payable 160,000
earnings, January 1 3,350,000 Net sales and other revenue Earnings from long-term contracts 6,680,000
15,000,000 Costs and expenses 10,000,000 Income tax expense Costs and expenses 5,180,000
2,100,000
27,900,000 27,900,000 All receivables on long-term contracts are considered to be collectible within 12 months.
During the year, estimated tax payments of P450,000 were charged to prepaid taxes.
The accounts receivable included PI,000,000 due from a' customer and payable in The entity has not recorded income tax expense. The tax rate is 30%. On December 31,
quarterly installments of Pl25,000. The last payment is due December 30, 2016. During 2014, what amount should be reported as
the year, estimated tax payment of P600,000 was charged to income tax expense. The
income tax rate is 30%. On December 31, 2014, what amount should be reported as 63. Total current assets?
A. 4,100,000 C. 5,700,000
60. Total current assets? B. 5,000,000 D. 6,150,000
A. 5,055,000 C. 5,555,000
B. 5,530,000 D. 6,030,000 64. Total noncurrent liabilities?
A. 1,620,000 C. 2,320,000
61. Total current liabilities? B. 1,780,000 D. 2,480,000
A. 2,700,000 C. 3,450,000
B. 3,300,000 D. 4,050,000 65. Total retained earnings?
A. 1,950,000 C. 2,400,000
62. Retained earnings? B. 2,110,000 D. 2,560,000
Current assets, noncurrent liabilities, retained earnings Multinational Company provided the following balances on December 31,2014:
Questions 1 thru 3 are based on the following information. P1 © 2014 Mint Company Accounts payable 500,000 Accrued taxes 100,000 Ordinary share
provided the following account balances on December 31,2014 which had been adjusted capital 5,000,000 Dividends - ordinary share 1,000,000 Dividends
except for income tax expense: - preference share 500,000 Mortgage payable (P500,000 due in
Cash 600,000 Accounts receivable 3,500,000 Cost in excess of six months) 4,000,000 Note payable, due January 31, 2016
billings on long-term contracts 1,600,000 Billings in excess of cost 2,000,000
on long-term contracts 700,000 Share premium 500,000 Preference share capital 3,000,000
Prepaid taxes 450,000 Premium on note payable 200,000 Income summary - credit
Property, plant, and equipment, at carrying amount 1,510,000 balance 4,000,000 Retained earnings - January 1 2,500,000
Note payable - noncurrent 1,620,000 Share capital 750,000 Unamortized issue cost on note payable 50,000 Unearned rent
Share premium 2,030,000 Retained earnings unappropriated income 150,000
900,000 Retained earnings restricted for note payable 160,000
Earnings from long-term contracts 6,680,000 Costs and expenses 69. What is the amount of noncurrent liabilities on
5,180,000 December 31, 2014? A. 5,500,000 C. 5,700,000
B. 5,650,000 D. 6,200,000
All receivables on long-term contracts are considered to be collectible within 12 months.
During the year, estimated tax payments of P450,000 were charged to prepaid taxes. 70. What is the amount of retained earnings on December
The entity has not recorded income tax expense. The tax rate is 30%. On December 31, 2014? A. 1,000,000 C. 5,000,000
31,2014, what amount should be reported as B. 2,500,000 D. 6,500,000

66. Total current assets? 71. What is the total shareholders' equity on December 31, 2014?
A. 4,100,000 C. 5,700,000 A. 8,500,000 C. 13,500,000
B. 5,000,000 D. 6,150,000 B. 9,500,000 D. 15,000,000

67. Total noncurrent liabilities?


A. 1,620,000 C. 2,320,000 ANSWER EXPLANATION
B. 1,780,000 D. 2,480,000
1. Answer is (C).
68. Total retained earnings? Cash and cash equivalent 700,000 Trade accounts & other
A. 1,950,000 C. 2,400,000 receivables (1,200,000 – 260,000) 940,000 Inventories
B. 2,110,000 D. 2,560,000 (600,000 + 200,000) 800,000 Total current assets 2,440,000

Noncurrent liabilities, retained earnings & shareholders’ equity 2. Answer is (C).


Questions 1 thru 3 are based on the following information. P1 © 2014 Cash (1,500,000 – 500,000) 1,000,000 Trade and other
receivables 3,000,000 Inventory (2,000,000 – 200,000) asset.
1,800,000 Total current assets 5,800,000
6. Answer is (A)
3. Answer is (C) Cash (3,500,000 + 500,000) 4,000,000
Inventory 1,000,000 Financial assets (2,000,000 - 500,000) 1,500,000
Accounts receivable 1,200,000 Inventories (800,000 - 200,000) 600,000
Prepaid insurance 100,000 Prepaid expenses ( 150,000 - 50,000) 100,000
Financial assets held for trading 200,000 Total current assets 6,200,000
Cash 300,000
Total current assets 2,800,000 7. Answer is (A)
In the absence of statement to the contrary, financial assets at fair value through Cash 4,300,000
other comprehensive income shall be classified as noncurrent. PAS 1 and PAS 12 Accounts receivable 5,000,000
provide that deferred tax asset is a noncurrent asset. The bank overdraft is Allowance for doubtful accounts ( 500,000)
classified as current liability. Inventory (4,000,000 + 2,000,000) 6,000,000
Total current assets 14,800,000
4. Answer is (D). The selling price of the unsold goods out on consignment is excluded from accounts
Accounts receivable 1,600,000 receivable but the cost of the goods shall be included in inventory. The cost of goods
Financial assets at fair value through profit or loss 500,000 out on consignment is P3,000,000 divided by 150% or P2,000,000.
Cash 1,100,000
Inventory 3,000,000 8. Answer is (C)
Prepaid expenses 100,000 Cash (2,000,000-500,000) 1,500,000
Equipment held for sale 1,800,000 Notes receivable 1,200,000
Total current assets 8,100,000 Note receivable discounted (700,000)
The financial assets at amortized cost shall be classified as noncurrent. Financial Accounts receivable - unassigned 3,000,000
assets at amortized cost include investment in bonds and other debt instruments. Accounts receivable - assigned 800,000
Allowance for doubtful accounts (100,000)
5. Answer is (A) Inventory (2,800,000-600,000) 2,200,000
Cash 200,000 Total current assets 7,900,000
Prepaid rent 100,000 The equity of the assignee in assigned accounts shall not be offset against the
Inventory 800,000 assigned accounts receivable but included in current liabilities.
Short-term investments 300,000 The note receivable discounted should be deducted from the total notes receivable
Accounts receivable 350,000 with disclosure of contingent liability.
Total current assets 1,750,000
The sinking fund and the investment in associate should be classified as noncurrent 9. Answer is (D).
Cash 2,000,000 Notes receivable 1,200,000 Note receivable Cash 300,000
discounted (700,000) Accounts receivable - unassigned 3,000,000 Total current assets 2,800,000
Accounts receivable - assigned 800,000 Allowance for doubtful In the absence of statement to the contrary, financial assets at fair value through
accounts (100,000) Inventory (2,800,000 – 600,000) 2,200,000 Total other comprehensive income shall be classified as noncurrent. PAS 1 and PAS 12
current assets 8,400,000 provide that deferred tax asset is a noncurrent asset. The bank overdraft is
classified as current liability.
10. Answer is (C)
Cash on hand (130,000 - 20,000 - 10,000) 100,000 13. Answer is (D).
Cash in bank (370,000 - 70,000) 300,000 Cash (3,700,000 + 300,000 -2,000,000) 2,000,000 Accounts receivable
Accounts receivable (1,900,000 + 50,000 + 20,000) 1,970,000 (1,500,000 - 100,000) 1,400,000 Allowance for doubtful accounts (200,000-
Allowance for doubtful accounts ( 150,000) 100,000) ( 100,000) Inventory (2,000,000 - 150,000) 1,850,000 Prepaid
Advances to employee 10,000 insurance (300,000-50,000) 250,000 Total current assets 5,400,000
Inventory (2,000,000 + 500,000 undelivered) 2,500,000
Prepaid expenses 100,000 14. Answer is (D).
Total current assets 4,830,000 Cash (300,000-100,000 overdraft) 200,000 Accounts receivable 350,000
Inventory 580,000 Prepaid expenses 120,000 Land classified as held for sale
Cost of undelivered goods (750,000/ 150%) 500,000 1,000,000 Total current assets 2,250,000
The undelivered checks of P300,000 should be adjusted by debiting cash and
11. Answer is (A) crediting accounts payable.
Cash 3,200,000
Accounts receivable 1,420,000 15. Answer is (A)
Allowance for uncollectible accounts (120,000) Cash (3,500,000 + 500,000) 4,000,000
Receivable from employees 240,000 Financial assets (2,000,000 - 500,000) 1,500,000
Claim receivable 200,000 Inventories (800,000 - 200,000) 600,000
Inventory 2,800,000 Prepaid expenses ( 150,000 - 50,000) 100,000
Total current assets 7,740,000 Total current assets 6,200,000
The advances to subsidiary shall be classified as noncurrent. The deferred charges
are considered noncurrent because technically these expire in more than one year
16. Answer is (B).
from the end of reporting period.
Current assets per book 6,500,000
Deferred tax asset (400,000)
12. Answer is (C)
Outstanding checks (70,000)
Inventory 1,000,000
Customers’ deposit 50,000
Accounts receivable 1,200,000
Overstatement of accounts receivable (750,000)
Prepaid insurance 100,000
Understatement of inventory (750,000 / 150%) 500,000
Financial assets held for trading 200,000
Total current assets 5,830,000 Accounts payable 550,000
Unsecured note payable 4,000,000
17. Answer is (D). Accrued expenses 350,000
Cash (3,700,000 + 300,000 -2,000,000) 2,000,000 Accounts receivable Senior bonds payable 5,000,000
(1,500,000 - 100,000) 1,400,000 Allowance for doubtful accounts (200,000- Total current liabilities 9,900,000
100,000) ( 100,000) Inventory (2,000,000 - 150,000) 1,850,000 Prepaid
insurance (300,000-50,000) 250,000 Total current assets 5,400,000 24. Answer is (A)
Accounts payable 1,900,000
18. Answer is (C) Dividends payable 500,000
Liabilities 1,800,000 Share capital 5,000,000 Retained earnings Income tax payable 900,000
(P2,500,000 less dividend P500,000) 2,000,000 Total liabilities and
Note payable 600,000
shareholders' equity 8,800,000
Total current liabilities 3,900,000
Under PAS 1 and PAS 12, a deferred tax liability should be classified as noncurrent.
19. Answer is (B) In the absence of any contrary statement, the bonds payable plus the premium on
Liabilities 2,000,000 Share capital 7,500,000 Retained earnings bonds payable should be classified as noncurrent. The dividends payable and
(8,200,000 - 6,400,000 - 300,000) 1,500,000 Total liabilities and income tax payable are normally classified as current.
shareholders' equity 11,000,000
25. Answer is (B)
20. Answer is (B)
Accounts payable (4,000,000 + 100,000) 4,100,000
Adjusted total assets (4,375,000- 120,000-42,000) 4,213,000
Accrued expenses 1,500,000
Credit balances in customers' accounts 500,000
21. Answer is (C).
Estimated liability for coupons 600,000
Accounts payable 5,000,000 Bonds payable, due December 30,
2015 10,000,000 Note payable-bank 4,000,000 Total current Total current liabilities 6,700,000
liabilities 19,000,000 The stock dividend payable is not an accounting liability but presented as part of
shareholders' equity as an addition to share capital. The claims for increase in
wages and allowance should be disclosed as contingent liability.
22. Answer is (A)
Accounts payable 1,500,000
26. Answer is (A).
Bonds payable 2,500,000 Trade accounts payable, including cost of goods (1,350,000 –
Discount on bonds payable ( 300,000) 150,000) 1,200,000 Accrued taxes payable 125,000 Customers'
Dividends payable 800,000 deposit 100,000 Bank overdraft 55,000 Accrued electric and power
Total current liabilities 4,500,000 bills 60,000 Total current liabilities 1,540,000
23. Answer is (B) 27. Answer is (C)
The 10% note payable is classified as noncurrent. PAS 1, paragraph 73, provides expense (6,000,000 x 30%) 1,800,000 Estimated tax payment (1,000,000)
that if an entity has the discretion to refinance or roll over an obligation for at least Income tax payable 800,000
twelve months after the reporting period under an existing loan facility, the The interest on the bonds payable is payable annually on June 30. Thus, there is an accrued
obligation shall be classified as noncurrent, even if it would otherwise be due within interest payable from July 1 to December 31, 2014 or six months.
a shorter period.
The 12% note payable is classified as current. PAS 1, paragraph 72, provides that 31. Answer is (C).
an obligation that matures within one year from the end of reporting period is Employee income taxes withheld 900,000
classified as current even if it is refinanced on a long-term basis after the reporting
Cash overdraft 1,300,000
period and before issuance of the financial statements. The 12%note payable is Accounts receivable with credit balance 750,000
refinanced on March 1,2015 and therefore classified as current. Estimated warranty liability 500,000
Estimated damages payable 1,500,000
28. Answer is (B). Employee income taxes withheld 900,000 Cash overdraft 1,300,000 Accounts receivable
Current portion of note payable (2,000,000 – 1,500,000) 500,000 with credit balance 750,000 Estimated warranty liability 500,000 Estimated damages
payable 1,500,000 Accounts payable 3,000,000 Accrued interest on bonds payable from
29. Answer is (D). The refinancing occurred on February 15, 2015, which is after the end October 1 to
of the reporting period and before issuance of the 2014 financial statement. Thus, December 31,2014 (5,000,000 x 12% x 3/12) 150,000 Total current liabilities 8,100,000
the note payable is classified totally as current. Accounts payable 3,000,000 Accrued interest on bonds payable from October 1 to
December 31,2014 (5,000,000 x 12% x 3/12) 150,000 Total current liabilities 8,100,000
30. Answer is (B). The bonds will be paid over 5 years because the semiannual payment is P500,000. Since the
Accounts payable 500,000 Accrued expenses 300,000 Dividends payable last bond will be paid on October 1,2020, the first bond will be paid on April 1,2016.
(100,000x7) 700,000 Accrued interest payable (5,000,000 x 8% x 6/12) 200,000 Accordingly, there is no currently maturing bond in 2014.
Income tax payable 800,000 Total current liabilities 2,500,000 Current tax

The bonds will be paid over 5 years because the semiannual payment is P500,000. Since the last bond 35.
Answer is (A)
will be paid on October 1,2020, the first bond will be paid on April 1,2016. Accordingly, 33. Answer is (B).
there is no currently maturing bond in 2014. Accounts payable 500,000
Accrued expenses 300,000
32. Answer is (C). Dividend payable (100,000 x 7) 700,000
Accounts payable 6,500,000 Bank note payable - 10% 3,000,000 Interest payable Accrued interest payable (5,000,000 x 8% x 6/12) 200,000
150,000 Mortgage note payable 2,000,000 Bonds payable-due June 30, 2015 Income tax payable (30% x 6,000,000 – 1,000,000) 800,000
4,000,000 Total current liabilities 15,650,000 Total current liabilities 2,500,000
34. Answer is (C). Accounts payable (1,250,000 + 200,000) 1,450,000
Current assets: Accrued expenses 250,000
Cash 5,000,000 Note payable-bank 500,000
Accounts receivable 20,000,000 Advances from customers 100.000 2,300,000
Allowance for doubtful accounts ( 1,000,000) Working capital 1,500,000
Merchandise inventory 13,000,000
Prepaid insurance 2,500,000 39,500,000 Current liabilities: 38. Answer is (A)
Accounts payable 8,000,000
Wages payable 2,000,000 39. Answer is (D).
Short-term note payable 3,000,000 13,000,000 Working capital 10% note payable issued on October 1, 2013 maturing October 1,2015
26,500,000 2,000,000 12% note payable issued on March 1, 2013 maturing on
March 1, 2015 4,000,000 Noncurrent notes payable 6,000,000
36. Answer is (B)
Current assets: 40. Answer is (D).
Cash in general checking account 500,000 Current assets - January 1, 2014 240,000 Property, plant and equipment-
Cash held to pay value added taxes 300,000 January 1, 2014 1,600,000 Total assets - January 1,2014 1,840,000 Current
Accounts receivable 2,100,000 liabilities (240,000-92,000) ( 148,000). Noncurrent liabilities ( 580,000)
Shareholders' equity-January 1, 2014 1,112,000
Inventory 1,500,000
Net income in 2014 64,000 Shareholders' equity - December 31,2014 1,176,000
Prepaid insurance 300,000 Current assets - December 31, 2014(130,000 + 92,000) 222,000 Property, plant
Equipment classified as held for sale 200,000 4,900,000 and equipment - December 31,2014 1,700,000 Total assets - December
Current liabilities: 31,2014 1,922,000 Current liabilities-December 31,2014 ( 130,000) Noncurrent
Accounts payable 1,400,000 Iiabilities-December31, 2014 (SQLEEZE) ( 616,000) Shareholders' equity -
Note payable - due July 2015 800,000 December 31, 2014 1,176,000
Salaries payable 400,000
Value added taxes payable 300,000 2,900,000 41. Answer is (B).
Working capital 2,000,000 Share capital 5,000,000
Retained earnings 1,000,000
37. Answer is (B). Treasury shares (750,000)
Current assets: Adjusted shareholders’ equity 5,250,000
Cash (300,000 + 500,000 + 200,000) 1,000,000
Accounts receivable (800,000 + 100,000) 900,000 42. Answer is (B)
Inventory 1,650,000 Share capital 5,000,000
Prepaid expenses 250,000 3,800,000 Share premium 2,000,000
Current liabilities: Retained earnings 500,000
Treasury shares, at cost ( 300,000) Reported net assets 8,750,000 Treasury shares ( 250,000)
Total shareholders' equity 7,200,000 Adjusted net assets 8,500,000

Cash 600,000 46. Answer is (C).


Financial at assets at fair value (1,000,000 - 3 00,000) 700,000 Net assets per book 8,750,000 Treasury stock (250,000)
Accounts receivable 3,500,000 Adjusted net assets 8,500,000
Inventory 1,500,000
Total current assets 6,300,000 47. Answer is (D).
Cash in bank (5,000,000 + 500,000) 5,500,000
43. Answer is (C) Petty cash (50,000 - 10,000) 40,000
Share capital 15,000,000 Notes receivable 4,000,000
Share premium 5,000,000 Accounts receivable (6,000,000 + 1,500,000) 7,500,000
Retained earnings unappropriated 6,000,000 Inventory 3,000,000
Retained earnings appropriated 3,000,000 Bond sinking fund 3,000,000
Revaluation surplus 4,000,000 Debit balances in accounts payable 1,000,000
Cumulative translation adjustment - credit 1,500,000 Actuarial Total current assets 24,040,000
loss on defined benefit plan ( 1,000,000) Treasury shares, at The bank overdraft is not netted against the cash in bank but should be classified as
cost ( 2,000,000) Total shareholders' equity 31,500,000 current liability. The customers' credit balances are not netted against accounts
The credit in the cumulative translation adjustment account is a receivable but should be classified as current liability. The bond sinking fund is
translation gain. If the cumulative translation adjustment account has classified as current asset because the bond payable is already classified as
debit balance, it is a translation loss. current liability. The classification of the fund should parallel the classification of the
related liability.
44. Answer is (B)
Sales 10,000,000 Total expenses ( 7,800,000) Net income 48. Answer is (D).
2,200,000 Retained earnings - January 1 1,000,000 Bank overdraft 500,000
Dividends ( 700,000) Retained earnings – December 31 Credit balances in accounts receivable 1,500,000
2,500,000 Accounts payable (7,000,000 + 1,000,000) 8,000,000
Notes payable 4,000,000
Preference share capital 2,000,000 Ordinary share capital Bond payable 3,000,000
3,000,000 Share premium 1,000,000 Retained earnings Accrued expenses 2,000,000
2,500,000 Treasury shares at cost ( 500,000) Total Total current liabilities 19,000,000
shareholders' equity 8,000,000 The debit balances in suppliers' accounts are not netted against accounts payable
but should be classified as current asset.
45. Answer is (D)
49. Answer is (C). 51. Answer is (A).
Cash in bank (1,650,000 - 250,000) 1,400,000 Cash 600,000 Accounts receivable 1,400,000 Total current assets 2,000,000
Accounts receivable (1,600,000 + 100,000) 1,700,000
Allowance for doubtful accounts ( 100,000) Accounts receivable 1,650,000 Noncurrent portion (125,000 + 125,000)
Advances to employee 50,000 ( 250,000) Current portion 1,400,000
Receivable from officers 300,000 Total retained earnings 1,330,000
Advances to suppliers 200,000 The debit balance in the foreign currency translation adjustment is a component of
Inventory (1,900,000 + 500,000) 2,400,000 other comprehensive income and a deduction from total shareholders' equity
Prepaid expenses 100,000 because it is a translation loss.
Debit balance in accounts payable 50,000
Total current assets 6,100,000 53. Answer is (A).
The customer check marked NSF should be reverted to accounts receivable. Cash 5,000,000
The cash deposit with court is classified as noncurrent. Accounts receivable (8,000,000 – 2,000,000) 6,000,000
The cost of undelivered inventory should be included in inventory. The amount is Inventory 6,000,000
computed as P600,000 divided by 120% or P500,000. Total current assets 17,000,000

50. Answer is (B). 54. Answer is (C).


Accounts payable (2,450,000 + 50,000) 2,500,000 Income before tax 5,000,000
Interest payable 150,000 Income tax (1,500,000)
Income tax payable 300,000 Net income 3,500,000
Mortgage payable - current portion (2,000,000 / 4) 500,000 Retained earnings – January 1 5,000,000
Total current liabilities 3,450,000 Total 8,500,000
The debit balance in accounts payable is not netted against accounts payable but
should be classified as current asset. 55. Answer is (D).
The money claim of the union pending final decision should be disclosed as contingent Cash (300,000-100,000 overdraft) 200,000 Accounts receivable 350,000
Inventory 580,000 Prepaid expenses 120,000 Land classified as held for sale
liability. 1,000,000 Total current assets 2,250,000
The prepaid taxes of P300,000 actually represent the current tax expense for 2014 and
The undelivered checks of P300,000 should be adjusted by debiting cash and crediting accounts
therefore should be charged to income tax expense. 700,000 Retained earnings-January 1 630,000
payable.
52. Answer is (B).
Revenue 3,600,000 Expenses (2,600,000) Income before income 56. Answer is (C).
tax 1,000,000 Income tax (30% x 1,000,000) ( 300,000) Net income Share capital 250,000 Share premium 1,500,000 Retained earnings
830,000 Total shareholders’ equity 2,580,000 Net sales and other revenue 15,000,000
Costs and expenses (10,000,000)
57. Answer is (A). Income before income tax 5,000,000
Cash 675,000 Accounts receivable 2,195,000 Inventory 2,185,000 Total Income tax (30% x 5,000,000) (1,500,000)
current assets 5,055,000 Accounts receivable 2,695,000 Noncurrent portion Net income 3,500,000
(125,000x4) ( 500,000) Adjusted current portion 2,195,000 Retained earnings - January 1 3,350,000
Retained earnings - December 31 6,850,000
58. Answer is (A).
Accounts payable and accrued liabilities 1,800,000 Income tax payable 63. Answer is (C).
(1,500,000 - 600,000) 900,000 Total current liabilities 2,700,000 Cash 600,000 Accounts receivable 3,500,000 Cost in excess of billings on long
The estimated tax payment of P600,000 should be adjusted as follows: term contracts 1,600,000 Total current assets 5,700,000 The prepaid taxes of
Income tax payable 600,000
P450,000 represent the tax expense for 2014.
Income tax expense 600,000
64. Answer is (A).
59. Answer is (B).
Note payable-noncurrent 1,620,000
Net sales and other revenue 15,000,000
The billings in excess of cost on long term contracts account is a current liability.
Costs and expenses (10,000,000)
Income before income tax 5,000,000
65. Answer is (B).
Income tax (30% x 5,000,000) (1,500,000)
Earnings from long term contracts 6,680,000
Net income 3,500,000
Costs and expenses (5,180,000)
Retained earnings - January 1 3,350,000
Income before income tax 1,500,000
Retained earnings - December 31 6,850,000
Income tax (30% x 1,500,000) ( 450,000)
Net income 1,050,000
60. Answer is (A).
Cash 675,000 Accounts receivable 2,195,000 Inventory 2,185,000 Total Retained earnings unappropriated 900,000
current assets 5,055,000 Accounts receivable 2,695,000 Noncurrent portion Retained earnings restricted 160,000
(125,000x4) ( 500,000) Adjusted current portion 2,195,000 Total retained earnings 2,110,000
61. Answer is (A).
Accounts payable and accrued liabilities 1,800,000 Income tax payable 66. Answer is (C).
(1,500,000 - 600,000) 900,000 Total current liabilities 2,700,000 Cash 600,000 Accounts receivable 3,500,000 Cost in excess of billings on long
The estimated tax payment of P600,000 should be adjusted as follows: term contracts 1,600,000 Total current assets 5,700,000 The prepaid taxes of
Income tax payable 600,000 P450,000 represent the tax expense for 2014.
Income tax expense 600,000
67. Answer is (A).
62. Answer is (B). Note payable-noncurrent 1,620,000
The billings in excess of cost on long term contracts account is a current liability.

68. Answer is (B).


Earnings from long term contracts 6,680,000
Costs and expenses (5,180,000)
Income before income tax 1,500,000
Income tax (30% x 1,500,000) ( 450,000)
Net income 1,050,000
Retained earnings unappropriated 900,000
Retained earnings restricted 160,000
Total retained earnings 2,110,000

69. Answer is (B).


Mortgage payable (4,000,000 - 500,000) 3,500,000 Note payable due January 31,2016
2,000,000 Unamortized issue cost ( 50,000) Premium on note payable 200,000 Total
noncurrent liabilities 5,650,000 Under PFRS 9, the unamortized issue cost is a deduction
from the related financial liability.

70. Answer is (C).


Retained earnings - January 1 2,500,000 Net income 4,000,000 Dividends -
ordinary share (1,000,000) Dividends - preference share (500,000) Retained
earnings - December 31 5,000,000
The net income is derived from the credit balance of the income
summary account. Needless to say, if the income summary account
has debit balance, it is a net loss.

71. Answer is (C).


Preference share capital 3,000,000
Ordinary share capital 5,000,000
Share premium 500,000
Retained earnings 5,000,000
Total shareholders' equity 13,500,000

You might also like